You are on page 1of 31

ELECTRIC CIRCUITS

Choose the correct alternative.


Only ONE choice is correct.
1. Identify the statement, which is not a necessary condition for an electric circuit.
a) It is always a closed path.
b) It must consist of at least one loop.
c) It must consist of a battery of cell or any other source of e.m.f.
d) None of these

2. Choose the incorrect statement about the resistor.


a) It always opposes the flow of current.
b) Voltage is always dropped across a resistor,
c) It always dissipates electric power into heat.
d) None of these

3. Two resistances are said to be in series if


a) equal current flows in each in the same direction.
b) equal current flows in each, may be in the opposite direction.
c) same current flows in each resistor and there no other option for the current to flow.
d) None of these

4. two resistances are said to be in parallel if


a) equal voltage is dropped across each resistor.
b) current is equally divided between the two resistors.
c) current division occurs between the two resistors.
d) None of these

5. Mark the incorrect statement about the network of resistors. In a network resistors.
a) resistance are always in series
b) resistances are always in parallel
c) resistances are either in series or in parallel
d) none of these
6. The electromotive forced of a battery is defined as
a) the potential difference across its terminals.
b) its ability to move electrons around the circuit.
c) the work done to move a unit positive charge around the closed circuit.
d) all the above.

7. Choose the correct statement about an ideal battery


a) It has no internal resistance
b) No energy is dissipated inside it
c) Voltage available to the external circuit is equal to its emf.
d) all the above

8. Choose the correct statement related to the battery delivering current to an electric circuit.
a) The current always comes out of its positive terminal
b) The current always enters through its negative terminal
c) The current leaving and entering the battery are always equal
d) All the above.

9. Across a real battery


a) voltage is always less than its emf
b) voltage is always more than its emf
c) voltage is more or less than its emf
d) none of these

10. Identify the statement which is not true for Kirchhoff’s Current Law
a) It is based on conservation of charge
b) It is necessary condition, which must be satisfied in any electrical network
c) It is applicable at a junction where minimum three branches must be meeting
d) None of these

11. Identify the statement, which is not true for Kichhoff’s Voltage Law.
a) It is based on energy conservation.
b) It is applied along a closed path.
c) The loop along which it is applied must contains both active and passive elements
d) None of these

12. Mark the incorrect statement related to Kichhoff’s laws.


a) According to Kirchhoff’s current Law total current approaching a junction is equal to the total current leaving
the junction
b) There is never an excess or deficiency of charge at a junction
c) Total voltage drop across a junction is equal to the voltage gain across the junction
d) none of these
Problems 13 to 19

The fig. 11.1 shows a network of five resistances and a battery.

13. The total resistance of the network across the battery is


a) 5 b) 6
c) 3.4 d) none of these

14. The current coming out of the battery is


a) 2.5 A b) 3 A
c) 4 A d) none of these

15. The voltage drop across the 10 resistor is


a) 6 V b) 9 V
c) 10 V d) none of these

16. The current in the 15 resistor is


a) 1 A b) 0.6 A
c) 1.5 A d) 0.4 A

17. The current through the 12 resistor is


a) 0.4 A b) 0.6 A
c) 0.75A d) 1 A

18. The power dissipated in the 3 resistor is


a) 25 W b) 37.5 W
c) 12 W d) 3.6 W

19. The maximum power is dissipated in the resistor of


a) 15  b) 12 
c) 4  d) 6 

Problems 20 to 25
The fig. 11.2 shows a network of four resistance and two batteries
along with two switches S1 and S2.

20. With the switch S1 and S2 opened


a) The current in the 4  resistor is 5 A
b) The current in the 1  resistor is zero
c) The current in the middle branch is I = A
d) All the above

21. With the switch S2 closed and S1 opened.


a) The current in the 4 resistor is zero.
b) The current in the 1  resistor is 5A
c) The current in the middle branch is I = A
d) All the above
22. With both the switches S1 and S2 closed.
a) The current in the 4 resistor is zero.
b) The current in the 1 resistor is zero.
c) The current in the middle branch is I = 0
d) None of these

23. With both the switches closed, the maximum power is dissipated in
a) 1 b) 2
c) 4 d) Middle branch

24. With both the switches closed


a) The 4 and 2 resistances in the left loop are in parallel
b) The 4 and 2resistances in the left loop are in series.
c) the left loop and the right loop are in series.
d) the left loop and right loop are in parallel

25.s The given circuit may be simplified as

a) b)

c) d)

Problems 26 to 33
The fig. 11.3 shows a network of five resistances and two batteries.

26. Choose the correct statement about the 3 resistance


10
a) Current due to 30 V battery is A
3
b) Current due to 15 V battery is 2.5 A
c) Total current is 4A
d) Both (a) and (b)

27. The current through 1 resistor flows


a) From C to D due to 30 V battery
b) Form D to C due to 15 V battery
c) From C to D as a result of both the batteries.
d) None of these
28. The current given by the 30 V battery to the circuit is
a) 4A b) 5A
10
c) A d) none of these
3

29. the current given by the 15 V batter to the circuit is


a) 2.5 A b) 3A
c) 2A d) none of these

30. The net current through the 30 V battery is


a) 3A b) 1A
c) 2A d) none of these

31. The net current through the 15 V battery is


a) zero b) 1 A
c) 3A b) none of these

32. Which of the batteries is getting charged.


a) 30 V b) 15 V
c) both d) none

33. The total electrical power consumed by the circuit is


a) 15 W b) 75 W
c) 105 W d) 90W

Problems 34 to 40
The fig. 11.4 shows a network of four resistances and three batteries.

34. Choose the correct alternative.


a) the potential distance VC – VF = 6V
b) No current flows in the branch CF.
c) Current flows in the branch from F to C.
d) Both (a) and (c).

35. Mark the incorrect statement


a) The current flowing in the left loop is independent of the right loop.
b) the current flowing in the right loop is independent of the left loop.
c) Both 30V and 15 V batteries do not produced current in the branch CF.
d) Both (a) and (b).

36. Choose the current statement related to the current in 4 resistor.


a) 4A current flows due to 30 V battery.
b) 1A current flows due to 6 V battery
c) Net current is 3A
d) None of these
37. Choose the incorrect statement related to the current in 1 resistor.
a) 2 A current flows due to 6V battery.
b) 5A current flows due to 15 V battery
c) Net current is 3A
d) None of these

38. Which of the battery is getting charged


a) Only 6V b) both 6V and 15 V
c) Only 15 V d) none

39. The current through the branch CF is


a) 4 A b) 3A
c) 7 A d) 1A

40. The electrical power dissipated as heat is


a) 207 W b) 123 W
c) 165 W d) none of these

41. The equivalent resistance between 1 and 6 of the network shown in fig. 11.5
is given by
r
a) r b)
2
r r
c) d)
3 4
42. Each side of the tetrahedron as shown in fig. 11.6 has a resistance r. its
equivalent resistance between the points C and D is given by
r
a) r b)
2
3r
c) d) none of these
2

Problems 43 to 45
The fig. 11.7 shows a network of resistance in which the point

43. The point which has the least potential is


a) A b) B
c) C d) D

44. The current through the 3 resistor is


a) 2 A from D to A b) 2A from A to D
c) 3.33 A from A to D d) 3.33 A from D to A

45. the current through the 4 resistor is


a) 0.5 A from B to C b) 0.5 A from C to B
c) 1A from C to B d) 1A from B to C

46. We have four identical cells each of emf 2 V internal resistance 1. Identify the correct arrangement of
these cells to get maximum current in the external resistance of 4.

a) b)

c) d)

47. It is desired to measure current and voltage across the 20 resistor with the help of an
ammeter and voltmeter. Identify the correct connection of ammeter (A) and voltmeter (V)
in the circuit.

a) b)

c) d)

48. The emf of a battery can be measured with


a) a voltmeter b) a potentiometer
c) en ideal voltmeter d) both (b) and (c)

49. Choose the incorrect statement related to a capacitor.


a) A capacitor stores energy.
b) A capacitor can cause current to flow in a circuit.
c) A capacitor can charge a battery.
d) None of these

50. When two capacitors are in series, then


a) same current flows through each capacitor
b) same charge flows through each capacitor
c) charge on the two capacitor may be different
d) all the above are correct

51. When two capacitor C1 and C2 are in series, then


a) The equivalent capacitance is C1 + C2
b) The equivalent capacitance is less than the lesser of C1 and C2
c) These cannot in the steady state condition
d) equal energy is also stored in each capacitor.
52. A capacitor
a) always allow time varying current pass through it.
b) only initially allows the steady current.
c) has infinite opposition to a steady current
d) only (a) and (c).

53. When two capacitor C1 and C2 are in parallel, then


a) the equivalent capacitance is always more than the larger of C 1 and C2.
b) Positively charged plates are joined together and negatively charged plates are also joined together.
c) Charge on each capacitor may be different but potential difference across each capacitor must be same.
d) all the above.

54. In the circuit shown in fig. 11.9


a) The capacitance between a and b is zero.
b) The capacitance between a and c is infinite.
C1C2
c) The capacitance between a and c is
C1 + C2
d) None of these

Problems 55 to 58
In the circuit shown in fig. 11.10, four capacitors are connected to a battery.

55. The equivalent capacitance of the circuit is


a) 25 F b) 6 F
c) 8.4 F d) none of these

56. The charge on the 5 F capacitor is


a) 60 C b) 24 C
c) 12 C d) 20 C

57. The potential difference across the 6 F capacitor is


a) 6V b) 4V
c) 5 d) none of these

58. The maximum energy is stored in the capacitor of


a) 10 F b) 6 F
c) 5 F d) 4 F
59. When a capacitor C is charged by a batter y of emf E through a resistor R,
then
a) it gets charged to a lesser magnitude of charge as compared to the situation
without resistance.
b) it stores less amount of electrostatic energy
c) it takes longer time to get completely charged
d) all the above are correct.

60. Choose the correct statement by comparing the charging of the capacitor as shown in fig. 11.12 with the
situation when there is no resistor
a) It gets charged to the same magnitude
b) Current through the battery never becomes zero.
c) Extra energy is dissipated in the resistor
d) All the above

61. Identify the correct definition of time constant


a) It is defined as the time during which the charge resis to 63 percent of its saturation value.
b) It is defined as the time during which the charge fall to 37 percent of it initial value.
c) It is defined as the time during which a capacitor has discharged by 63 percent .
d) All the above

62. In a discharging RC circuit, at t = 0, I0 = 10 A and at t = 20 s, the current reduces


to
I = 3.7A.Determine the value of t at which current becomes 5 A.
a) 6.93 s b) 13.86 s
c) 28 s d) none of these

63. The value of the time constant  in the circuit shown is


a) R1C b) R2C
 RR 
c)  1 2  C d) (R1 + R2) C
 R1 + R2 

64. the value of the time constant  in the circuit shown is


 RR   RR 
a)  1 2  ( C1 + C2 ) b)  1 2  ( R1 + R2 )
 R1 + R2   R1 + R2 
c) (C1 + C2) (R1 + R2) d) none of these
Choose the correct alternative.
MORE THAN ONE choice (s) is/are correct.
1. Choose the correct statement (s) related to battery in an electric circuit.
a) A battery always delivers electric current to a circuit.
b) In a circuit, current always comes out of its positive terminal, and enters into its negative terminal.
c) A battery always delivers energy to the circuit.
d) None of these

2. Mark the correct statement (s) related to a real battery connected in an electrical network.
a) There is always a voltage gain across the battery.
b) There is always a voltage drop across the battery
c) There is either a voltage drop or gain across a battery
d) None of these

3. Two resistance R1 and R2 (>R1)are connected in parallel, then the equivalent resistance of the combination.
a) is always less than R1 b) may be more than R2
c) may lie in between R1 and R2 d) always lie in-between R1 and R2

4. A uniform wire of resistance R is bent in the form of a ring. Four points A, B, C


and D are marked on the ring as shown in fig. 11.15. A battery of emf E may be
connected across the ring in two ways:
Case I : Battery is connected between B and D.
Case II: Battery is connected between A and B.
Choose the correct statement (s)
a) In each case same current flows out of the battery.
b) In case I more current comes out of the battery
c) In case I more current comes out of the battery
d) In case I the minimum current comes out of the battery

Problems 5 to 8
The fig. 11.16 shows a network of resistors and a battery. If 1A current
flows through the branch CF, then answer the following questions.

5. The current through


a) branch DE is 1A b) branch BC is 2A
c) branch BG is 4A d) branch HG is 6A

6. The emf E of the battery is


a) 24 V b) 12 V
c) 18 V d) 6 V

If a zero resistance wire is connected in parallel to branch CF

7. The current through


a) branch DE is zero b) branch BC is zero
c) branch BG is 0.5 A d) branch AB is 1.5 A

8. The emf E of the battery is


a) 9 V b) 6.6 V
c) 5.25 V d) 10.5 V

Problems 9 to 10
In the circuit as shown in fig. 11.18, if no current flows through the branch
CD, then answer the following questions.

9. Choose the correct statement (s).


a) Potential difference VF – VE = 0.
b) Potential difference VC – VF = 0.
c) 15 V battery does not deliver current
d) No current flows in the branch DE.

10. Identify the correct statement (s)


a) Current in 4  resistor is 5 A
b) Current in 3 resistor cannot be obtained
c) The unknown emf E of the battery is 45 V
d) The unknown emf E of the battery is 30 V
Problems 11 to 13
A battery of emf E with internal resistance r is connected with a generator through
a resistance R as shown in fig. 11.19.

11. In order to charge the battery, the output voltage V of the generator
a) must be greater than E.
b) must be at least twice of E.
c) must be very large as compared to E.
d) must not be more than E.

12. The charging current U through the circuit is given by


V V +F
a) b)
R+r R+r
V−E E−V
c) d)
R+r R +f

13. In a time t, the energy


a) stored in the battery is (E – Ir) It
b) energy dissipated as heat is I2 (R + r) t
c) energy supplied by the generator is VIt
d) energy supplied by the generator is (V – E) It.

Problems 14 to 19
The fig. 11.20 shows a network of resistances and a battery.

14. Identify the correct diagram (s) from the given four choices, which is
equivalent to the given network.

a) b)

c) d)

15. It the circuit shown in fig. 11.21, the current through


a) branch AB is I1 b) branch AD is I – I1
c) branch DC is I – I1 – I2 d) branch DC is I – I1 –
I2

16. Kirchhoff’s voltage law is applied along different loops in the given circuit,
Identify the correct independent equations.
a) I – 2I1 – I2 = 0 along ABDA
b) 6I + 17I1 + 29I2 = 0 along BCDB
c) 18I + 41I1 + 17I2 = 0 along ABCDA
d) 10I – 17I1 – 9I2 = 4 along ABCEA

17. In the given circuit, if the point C is assumed to have zero potential, i.e. VC = 0, then
a) the potential of point A is + 100V
b) the potential VB > VD
c) the potential VD > VB
d) the potential VA < 100V

18. Applying Kichhoff’s current law at the junction B, we get


V −V V − VB VB V −V V − VB VB
a) A B + D − =0 b) A B + D + =0
12 12 11 12 12 11
V − VA VB − VD VB V −V V V − VD
c) B + + =0 d) A B − B = B
12 12 11 12 11 12

19. Applying Kirchhoff’s current law at the junction A, we get


V − VA VD − VA VA V − VA VD − VA 100 − VA
a) B + − =0 b) B + + =0
12 12 10 12 12 10
V − VA VD − VA 100 + VA V − VA VD + VA VA − 100
c) B + − =0 d) B + =
12 12 10 12 12 10

Problems 20 to 25
The given fig. 11.22 shows a network opf resistances and a battery.

20. Identify the correct statement(s).


a) The circuit satisfies the condition of a balanced wheatstone bridge.
b) VB – VD = 0
c) VB – VD = 8V
d) no current flows in the branch BD

21. The current in the branch BD


a) is only due to 8 V battery b) is only due to 12 V battery
c) is the resultant effect of both the batteries. d) flows from D to B

22. the current in the branch AC


a) flows form C to A b) in only due to 12 V battery
c) includes the contribution of 8V battery also d) is only due to 8V battery

23. Which of the two batteries is getting charged?


a) 8V battery b) 12 V battery
c) None d) Cannot be said

24. Choose the correct statement (s)


a) The resistance of the circuit for 8V battery is 4.

b) The resistance of the circuit for 8 V battery is not defined.


c) The resistance of the circuit for 12 V battery is 4.
d) The resistance of the circuit for 12 V battery is not defined.

25. Choose the correct statement (s).


a) The current coming out of the 8V battery is 2A.
b) The current coming out of the 12V battery is 3A.
c) The current flowing in all the 4 branches is same.
d) The current flowing in the diagonally opposite branches is same.
Problems 26 to 28
Inside a super-conducting ring six identical resistors each of resistance R are
connected as shown is fig. 11.23

26. The equivalent resistance (s)


a) between 1&2, 2&3 , 3&1 are all equal
b) between 1&3 is zero.
c) between 1&3 is R/2.
d) between 1&3 is two times that between 1&2.

27. The equivalent resistance(s)


a) between 0&1, 0&2 and 0&3 are ll equal. b) between 0&1 is R.
c) between 0&1 is R/3 d) between 0&1 is zero.

28. Imagine a battery of emf E between the point 0 and 1, with its positive terminal connected
a) The current entering at 0 is equally divided into three resistance
b) The current in the other three resistance R12, R13, R23 is zero.
c) The resistance R02 and R03 have equal magnitudes of current while is resistance
R01 have different current
d) Potential V2 = V3 > V1.

Problems 29 to 31
The fig. 11.24 shows a tetrahedron, each side of which has a resistance r.

29. Choose the correct statement (s) related to the resistance between any two points.
a) RAB = RBD = RBC = RCD = RCA = RAD
b) RAB = RAC = RAD = RBD = RBC  RCD
c) RCD is the least,
d) RAB = RAC = RBC amd RCD = RAD = RBD

30. Choose the correct diagram (s), which shows two-dimensional equivalent of the tetrahedron,

a) b) c) d)

31. If a battery is connected between any two points of the tetrahedron, then identify the correct statement (s).
a) The potential of the other two points are always equal.
b) The always exists a branch through which no current flows.
c) The current coming out of the battery is each case is same.
d) None of these

32. In the circuit shown in fig. 11.25


a) the two cells are connected in series.
b) the potential difference between the points A and B cannot he zero.
c) the potential difference between A and B becomes zero when E 1r2 = E2r1.
d) when VA = VB, no current flows in the circuit.

33. Identify the correct statement (s) related to a galvanometer.


a) It measures current.
b) It is marked with positive and negative polarity.
c) the deflection in the galvanometer is proportional to the current.
d) Zero is marked at the middle of the scale.
34. Choose the correct statement related to the conversion of a galvanometer into ammeter.
a) By connecting a low resistance in parallel with galvanometer.
b) By connecting a low resistance in series with galvanometer.
c) By connecting a high resistance in series with galvanometer.
d) by connecting a high resistance in parallel with galvanometer.

35. Choose the statement (s) which are correct for the magnitude of shunt resistance.
a) Higher the resistance of galvanometer, larger the value of shunt resistance,
b) Lower the resistance of galvanometer, larger the value of shunt resistance.
c) Larger the range of ammeter, lower the value of shunt resistance
d) Larger the full scale deflection current, larger the value of shunt resistance.

36. Choose the correct statement (s) about an ammeter.


a) It measures current.
b) It is always connected across the branch through which is has to measure current.
c) It always measures the exact value of current flowing in the branch.
d) The resistance of an ideal ammeter is zero.

37. Choose the correct statement (s) related to the conversion of a galvanometer into voltmeter.
a) By connecting a low resistance in parallel with galvanometer.
b) By connecting a high resistance in parallel with galvanometer,
c) By connecting a high resistance in series with galvanometer.
d) by connecting a low resistance in series with galvanometer.

38. Choose the correct statement(s) related to the magnitude of external resistance connected to a galvanometer.
a) Higher the magnitude of galvanometer resistance , lower the magnitude of external resistance
b) Higher the magnitude of galvanometer resistance , higher the magnitude of external resistance
c) Larger the full scale deflection current, lower th value of external resistance.
d) Larger the range of voltmeter, larger the value of external resistance.

39. Identify the correct statement (s) related to the voltmeter.


a) It always measure the voltage across itself.
b) It always measure the voltage between the points across which it is connected.
c) A voltmeter with a maximum possible resistance is always preferred.
d) The resistance of an ideal voltmeter is infinite.

40. A potentiometer can be used to measure


a) emf of an unknown cell. b) internal resistance of a cell.
c) unknown resistance d) voltage drop across a resistor

41. The deflection in a galvanometer falls from 50 divisions to 20 division when a 12 shunt is applied,. The
resistance of the galvanometer is
a) 18  b) 36
c) 24  d) 8

42. A potentiometer of E0 volt is used to determine the internal resistance of unknown cell of E volt. A balance
point of the cell in the open circuit is obtained at length 0 . When a resistor R is connected in parallel with the
cell, then the balance point is obtained at . Choose the correct statement (s).
a) E0 must be more than E.
b) 0 is always more than .
c) 0 may be less than
d) In both the readings no current is coming out of the cell.

43. In the above problems if E0 = 2V, 0 = 75 cm and = 60 cm. Then, the emf of the cell is given by
(assuming length of potentiometer as 1m)
a) 3 V b) 1.5 V
8
c) V d) 1.6 V
3
44. In problem 43, the magnitude of external resistance R is
a) 12 b) 9 
c) 25  d) insufficient data

45. In problem 43, If R = 100  then internal resistance r of the cell is


a) 2.5  b) 2
c) 1.5  d) 8

Problems 46 to 49
The fig. 11.26 shows a circuit diagram with diagonally symmetric resistance.

46. Choose the correct statement (s) related to current in each branch.
a) I1 = I – I1
b) I1 = I3 + I2
c) I5 = I1
d) I2 = I4

47. Choose the correct statement related to the potential differences.


a) VA – VB = VB – VC b) VA – VB = VD – VC
c) VA – VD = VB – VC d) VA – VC = VD – VC

48. If r1 > r2, then choose the correct statement (s).


a) VB – VD > 0 b) VB – VD < 0
c) VD + VB = VA + VC d) VD + VB = 2VA – E

49. The equivalent resistance of the circuit is


(r + r ) r + r r
a) 1 2 0 1 2 b)
( r1 + r2 ) r0 + 2r1r2
r0 + r1 + r2 2r0 + r1 + r2

c)
( r1 − r2 ) r0 + r1r2 d)
( r1 − r2 ) r0 + 2r1r2
r0 + r1 − r2 2r0 + r1 − r2

50. In a series combination of two capacitors C1 and C2 (> C1). As shown in fig. 11.27

a) C2 stores more energy than C1 .


b) C1 stores more energy than C2.
c) Potential difference across C1 is more than that of C2.
d) Potential difference across C2 is more that that of C1.

51. Ina parallel combination two capacitors C1 and C2 (>C1). As shown in fig. 11.28
q C
a) The ratio of charges 1 = 1
q2 C2
q1 C2
b) The ratio of charges =
q2 C1
U1 C1
c) The ratio of energy stored =
U 2 C2
U1 C 2
d) The ratio of energy stored =
U 2 C1
52. When a changed capacitor is connected with an unchanged capacitor, then flows
a) The change always flows from the changed to the unchanged capacitor.
b) A steady state is obtained after which no further charge flow occurs.
c) The total potential energy stored in the capacitors remains conserved.
d) The charge conservation always holds true

53. In the above problem,


a) The uncharged capacitor always shares the less charge
b) The uncharged capacitor always share the more chare
c) The on with longer capacitance shares the more charge
d) The one with smaller capacitance share the more charge

54. In the problem 52


a) The electrostatic energy lost by the charged capacitor is equal to that gained by the uncharged capacitor.
b) The electrostatic energy lost by the charged capacitor is always more than that gained by the uncharged
capacitor.
c) The capacitors share energy is proportional
d) The capacitors share energy is increase proportion to their capacitance.

55. In the problem 52


a) the energy loss is proportional to the capacitance of the charged capacitor.
b) the energy loss is proportional to the capacitance of the uncharged capacitor.

c) the energy loss is independent of the capacitance of the uncharged capacitor.


d) none of these

Problems 56 to 59
The fig. 11.29 shows a diagonal symmetric arrangement of capacitors and a battery.

56. Identify the correct statements


a) Both the 4F capacitors carry equal charged in opposite sense.
b) Both the 4F capacitors carry equal charges in same sense.
c) VB – VD > 0
d) VD – VB > o

57. If the potential of C is zero, then


a) VA = +20 V b) 4(VA – VB) + 2(VD – VB) = 2VB
c) 2(VA – VD) +2 (VB – VD) = 4VD d) VA = VB + VD

58. The potential of the point B and D are


a) VB = 8V b) VB = 12V
c) VD = 8V d) VD = 12V

59. The value of charge q1, q2 and q3, as shown in the fig. 11.30 are
a) q1 = 32C; q2 = 24 C; q3 =-8C
b) q1 = 48C; q2 = 16C; q3 = +8C
c) q1 = 32C; q2 = 24C; q3 = +8C
d) q1 = 3C; q2 = 4C; q3 = + 2C

Problems 60 61
In the circuit as shown in fig. 11.31 the switch is closed at t = 0.

60. At the instant of closing the switch,


a) the battery delivers maximum current
b) no current flows through C
c) voltage drop across R2 is zero
d) the current through the battery decreases with time finally becomes zero.

61. A long time after closing the switch


a) voltage drop across the capacitor is E.
E
b) current through the battery is
R1 + R2
1  R2 E 2 
c) energy stores in the capacitor is c 
2  R1 + R2 
d) current through the capacitor becomes zero.

Problems 62 to 63
The charge across the capacitor in two different RC circuit 1 and 2 are
plotted as shown in fig. 11.32

62. Choose the correct statements(s) related to the two circuits.


a) Both the capacitors are charged to the same magnitude
b) The emf of calls in both the circuit s are equal.
c) The emf’s of the calls may bge different
d) The E1 is more than E2.

63. Identify the correct statement (s) related to the R1,R2,C1 and C2 of the two RC circuits.
a) R1 > R2 if E1 = E2 b) C1 < C2 if E1 = E2
R C
c) R1C1 > R2C2 d) 1  2
R2 C1

64. the instantaneous charge on a capacitors in two discharging RC circuits is


plotted with respect to time in fig. 11.33. Choose the correct statement(s).
R C
a) R1C1 > R2C2 b) 1  2
R2 C1
c) R1 > R2 if E1 = E2 d) C2 > C1 if E1 =E2
ANSWERS
Only ONE choice is correct.
1c 2b 3c 4d 5d 6c 7d 8d 9d 10c
11c 12d 13a 14b 15a 16d 17c 18c 19c 20d
21d 22d 23c 24b 25c 26c 27d 28a 29b 30a
31b 32d 33c 34a 35c 36b 37d 38a 39c 40b
41d 42d 43b 44a 45c 46a 47c 48d 49d 50d
51b 52d 53d 54d 55b 56d 57b 58b 59c 60d
61d 62b 63b 64b

MORE THAN ONE choice is correct.

1d 2d 3a 4cd 5ab 6b 7a 8a 9ad 10ac


11a 12c 13bc 14abc 15abd 16acd 17bd 18acd 19bd 20ac
21ad 22ab 23c 24ac 25abd 26ab 27ac 28ab 29a 30abcd
31abc 32ac 33cd 34a 35acd 36ad 37c 38acd 39abcd 40abcd
41a 42ab 43b 44d 45a 46abcd 47bc 48bcd 49b 50bc
51ac 52abd 53c 54bc 55b 56bc 57abcd 58bc 59c 60ac
61bc 62ac 63d 64ac

ELECTRIC CIRCUITS (SOLUTIONS)

Only ONE choice is correct.

1. c
A circuit need not contain a cell or a battery or any other source of emf. A charged capacitor can also drive
current in a circuit.

2. b
There is always a voltage across a resistor. But the voltage drop or gain depends on the direction of current
and the direction of movement.
• If we move in the direction to current voltage drop occurs.
• If we move in the opposite direction of current voltage again occurs .

3. c
Note that the same current does not mean the equal current .

4. d
Two resistance are said to be in parallel when both the terminals of the resistors are connected to each other
.

In fig (A) and (B), the 2 and 4 resistors are in parallel . But in fig. S 11.1 (C) , the 2 resistor is not in
parallel with 4 resistor, although current division occurs at junction A

5. d
Resistors may be arranged in any arbitrary manner not necessarily the series or parallel combination .

6. c
The electromotive force is defined for a non–conservative field. It is defined as the work done in moving a
unit positive charge around the closed circuit .
Wloop
E=
q
7. d
In the addition to three mentioned properties, the emf of an ideal battery does not changed with time.
8. d
When a battery delivers current .
• the current comes out of its positive terminal, and
• the current enters into its negative terminal
When a batteryis getting charged in a circuit .
• the current enters into its positive terminal, and
• the current comes out of its negative terminal.
But in either case, the total current coming out of it is equal to the total current entering into it.

9. d
The voltage across a real battery depends on the magnitude and direction of current .
• If I = O , voltage is equal to its emf.
• If the current enters into its positive terminal, then V > E .
• If the current comes out of its positive terminal, then V < E .

10. c
The Kirchhoff’s Current Law is applicable at any point in the circuit. But usually , we apply at a point where

at least three branches are meeting .

11. c

The Kirchhoff’s Votlage Law is applicable along a loop containing resistory only and containing cells or
batteries only.

12. d
All the statements are correct.

Problems 13 to 19

13. a
1 1 1 1
= + +  R1 = 2
R1 15 10 3
1 1 1
= +  R 2 = 3
R 2 12 4
 R = R1 + R 2 = 5

14. b
15
I= = 3A
5

15. a
Voltage across the 10 resistor is equal to the voltage across R1
i.e. V10 = I R1 = (2A) (3) = 6V
16. d
V 6
The current in the 15 resistor is I15 = 10 = = 0.4 A
15 15

17. c
 4  4
The current through the 12 resistor is I12 = I   = 3   = 0.75A
 12 + 4   16 

18. c
V32 V102 (6) 2
The power dissipated in the 3 resistor is P3 = = = = 12W
3 3 3

19. c
The maximum power is dissipated in 4 resistor .
V2
P4 = 4
4
(9)2
Since V4 = 15 − V10 = 9 V , therefore , P4 = = 20.25 W
4

Problems 20 to 25

20. d
When the switch S1 is closed and S2 is opened , the current flows only
in the first loop
30
 I1 = = 5A
4+2

Thus, all the choices are correct.

21. d
When the switch S2 is closed and S1 is opened, the current flows only
in the second loop .
15
 I1 = = 5A
1+ 2
Thus all the choices are correct

22. d
When both the switches are closed, the current flows through both the
loops. Since the middle branch
contains no resistance, therefore, the current in each loop is
independent of the presence of other .
Thus, I1 = 4A ; I2 = 5A
and , the middle branch current I = I1 + I2 = 10A
Hence, no option is correct.
23. c
Since equal current is flowing in all the resistances, therefore, maximum power is dissipated in the resistor
with maximum resistance, i.e. 4
 Pmax = I2 R = (5)2 (4) = 100W

24. b
• In the left loop both 4 and 2 resistances are in series because same current flow in each of them.
• The left loop and the right loop have no relation with each other .

25. c
Since potential difference across the two ends of the middle branch is
zero , therefore the two ends can be joined as one point as shown in
fig. S 11.6.
Problems 26 to 33

26. c
With the 30V battery only, as shown in fig. S 11.7
(A), the current through the 3 resistor is
I1 = 2A
With the 15V battery only, as shown in fig. S
11.7 (B), the current through the 3 resistor is
I2 = 2A
 Total current through 3 resistor is
I = I1 + I2 = 4A

27. d
In fig. S 11.7 (A), the current through 1 resistor flows from C to D , and it is equal to 2A. In fig. S 11.7
(B), the current through 1 resistor flows from D to C and it is equal to 3A .
Therefore, the net current flow from D to C , i.e.
I = 3–2 = 1A

28. a
The total current supplied by the 30V battery is 4A . It is obvious from the circuit shown in fig. S 11.7
(A)
29. b
The total current supplied by the 15V battery is 3A, as explained with circuit diagram shown in fig.
S 11.7 (B)

30. a
The contribution of 15V battery in the branch AB is equal to 1A (from B to A) .
The contribution of 30V battery in the branch AB is 4A (from A to B).
Hence , using superposition principle , net current through the 30V battery is
(4 – 1) = 3A from A to B.

31. b
The contribution of 15V battery in the branch DE is 3A (from E to D).
The contribution of 30V battery in the branch DE is 2A (from D to E).
Hence , the net current through the 15V battery is (3 – 2) = 1A from E to D.

32. d
Since both the batteries are delivering current, therefore, none of the
two is getting charged.

33. a
The total electrical power consumed is equal to the electrical power supplied, i.e.,
P = P1 + P2 = (30V) (3A) + (15V) (1A)
or P = 105 W

Problems 34 to 40

34. a
Obviously, the potential difference
VC − VF = 6V
The orientation of the 30V and 15V battery is such that each of them
tries to flow the current from C to F and since the magnitude of emf of
each battery is more tan 6V , therefore, each one of them is able to low
the current flow C to F.

35. c
As there is no resistance in the branch CF, therefore, the currents in the left and the right loop are
entirely independent of each other .

36. b
• In the left loop, 5A current flows in the clockwise sense due to 30V battery and 1A currents flows in
the anticlockwise sense due to 6V battery
• The net current is (5–1) = 4A (clockwise)

37. d
All the choices are correct.

38. a
Since only 6V battery is taking current from the circuit and the
other two batteries are delivering current to the circuit. Hence only 6V
battery is charged.

39. c
Obviously , from the circuit diagram shown in fig. S 11.10, current through the branch CF is 7A.

40. b
The electrical power dissipated as heat is
P = (4A)2 (4 + 2) + (3A)2 (1 + 2)
= 96 + 27 = 123W

41. b
The points 0, 1 and 4 have same potential , and the points 5 and 6 have same potential
. Therefore, the circuit may be reduced as shown in fig. S 11.11
r
Thus, R16 =
4

42. b
With respect to points C and D, the given circuit reduces to a wheatsone bridge, as
shown in fig. S 11.12. Thus, the branch AB can be removed and the equivalent
r
resistance between C and D is .
2

Problems 43 to 45

43. b
The potential of the various points are :
VA = 0 ; VB = –2V ; VD = –2 + 8 = 6V ; VC = 6 – 4 = + 2V

44. a
VD – VA = 6 – 0 = 6V
V − VA 6
I= D = = 2A (from D to A)
3 3

45. c
VC – VB = 2 – (–2) = 4V
V − VB 4
 I4 = C = = 1A (from C to B)
4 4

46. a
• To get maximum current in the external resistor , we have
m r
=
n R
where n is the number of cell in each row.
m is the number of rows.
m 1 1
here = =
n 4 4
Hence, all the four cells should be connected in series to get the maximum current in 4 resistor.
• The value of maximum current is
E mn (2V) (1)(4)
Imax = = = 1A
2 rR 2 (1)(4)

47. b
The arrangement shown in (b) is more close to the actual values.

48. d
The emf of a battery is measured when no current is flowing through it. This situation can occur only
with an ideal voltmeter or a potentiometer.

49. d
• A capacitor stores energy in electric field.
• A capacitor can cause current to flow in a circuit in the absence of an active source (cell or battery) or if the
opposing emf of the active source is less than the potential difference across the capacitor.

50. d
The charge an capacitors connected in series may be different if their initial charges are different .

51. b
C1C 2
• The quantity C = is less than both C1 and C2 .
C1 + C 2
• The energy stored in the capacitors are in inverse proportion to their capacitances.

52. d
• A capacitor does not allow the steady current to pass through it .

53. d
All the statements are correct and very obvious

54. d
The capacitance between a and b is infinite
The capacitance between a and c is C2 .

Problems 55 to 58

55. b
(10 + 5)(6 + 4)
C= = 6F
(10 + 5) + (6 + 4)

56. d
5  5 
q5 = q0 = (6) (10)   = 20C
5 + 10  5 + 10 

57. b
q5 20C
V5 = = = 4V
5F 5F

58. b
The equivalent capacitance of 10F and 5F is 15F
The equivalent capacitance of 4F and 6sF is 10F
1
Therefore , maximum energy is stored in the unit containing 4F and 6F capacitor, because C  for
C
same q.
In the unit itself, maximum energy is stored in the 6F capacitor because U  C for same V .

59. c
• The maximum charge attained by a capacitor is independent of the resistance in series with it .
• Therefore , the energy stored in the capacitor is also independent.
• Since the presence of series resistance increases the magnitude of time constant , therefore , it takes
longer time to get completely charged.

60. b
• The potential difference across the capacitor remains E .
• Current through the battery never becomes zero because the resistor provides an alternative path.
• The resistor dissipates energy as heat .

61. d
All the statements correctly explains the definition of time constant.

62. b
The current reduces to 50% of its initial value in a time equal to
t = 0.693 
or t = 0.693 (20) = 13.86 s

63. b
The equivalent resistance of the circuit
RR
is R = 1 2
R1 + R 2
 RR 
  = RC =  1 2 C
 R1 + R 2 

64. b
The given capacitors are in series and the resistances are in parallel .
 C C  R R 
Therefore ,  =  1 2  1 2  C
 C1 + C2  R1 + R 2 

MORE THAN ONE choice (s) is/are correct.

1. d

• A battery always delivers current to the circuit when no other energy source in the circuit exists.
• When a battery is being charged, then current enters through its positive terminal and comes out of its
negative terminal.
• During charging, the battery takes energy from the circuit.

2. d
Voltage drop and voltage gain across a battery depends on the
direction of movement across it.
For example, in fig. S 11.15 .

• Voltage drop occurs as we move from A to B , i.e.


VA + E + Ir = VB
• Voltage gain occurs as we move from B to A , i.e.,
VB – Ir + E = VA
• No voltage drop or gain occurs across the battery if
E – Ir = 0
3. a
 R2  R2
  R1  R1 because 1
 R1 + R 2  R1 + R 2

4. c & d
The problem may be visualized as a circuit in which a battery is supplying
current in two parallel resistances, as shown in fig. S 11.16 .

In case I : R1 = R2 , therefore
Req = R1 = R2

In case II : R2 < R1 , therefore, Req < R2


Thus , in case II, more current comes out of the battery ; and in case I, the minimum current comes out of
the battery, because the amximum possible value of Req is R1 or R2 .
Problems 5 to 8
5. a & b
• Since the branch CDE is in parallel with the branch CF and it has
equal resistance, therefore equal current also flows through it . i.e.
IDE = 1A

• The current coming from the branch BC is split up into CF and


CD, therefore,
IBC = 1 + 1 = 2A
• The resistance in parallel to branch BG is 3 and it carries a
current of 2A. Therefore, IBG = 1A

• Using Kirchhoff’s Current Law, IGH = 2A + 1A = 3A

6. b
The total resistance of the circuit is
R = 4
The current through the branch AB is IAB = 1 + 2 = 3A
 E = R IAB = 4(3) = 12 V

7. a
When a zero resistance wire is connected across the 2 resistor in
the branch CF, then no current flows through CD and the current
through BC becomes 1A
 2G  1
Thus, IBG =   (1A) = A
 6  3
1 4
and IAB = IBG + IBC = + 1 = A
3 3

8. a
The emf of the battery is E = IAB Req
(6) (2) 3 7
R eq = 2 + = 2+ =
6+2 2 2
 4  7  14
or E =    = = 4.67 V
 3  2  3
Problems 9 to 10

9. a & d
When current through the branch CD is zero, then
IDE = 0 = IEF
 VF – VE = 2IEF = 0
VC – VF = VD – VE = 15V

The current delivered by the 15V battery is equal and opposite to the current produced
in branch CD by E.

10. a & c
VC − VF 15
Since Vc – VF = 15V, therefore , ICF = = = 5A
3 3
Also, IBC = ICF = IFA = 5A
The emf of the unknown battery is E = (5 A) (4 + 3 + 2) = 45V

Problems 11 to 13

11. a
A battery gets changed only when output voltage of the charger is greater than the emf of the battery .

12. c
Applying KVL, we get
V – E – Ir – IR = 0
V−E
or I =
R+r

13. b & c
• Energy supplied by the generator is VI t
• Energy stored in the battery is EI t
• Energy dissipate as heat is I2 (R + r) t

14. a, b, c

• The (d) choice will become correct if the branch BD overlaps the branch AEC .
15. a, b, d
Applying Kirchhoff’s Current Law first at junction B, then at junction A
and finally at junction C.

16. a, c, d
• Applying Kirchhoff’s Voltage Law along the loop ABDA ,
we get –12I1 – 12I2 + 12 (I – I1) = 0
12I – 24I1 –12I2 = 0
or I – 2I1 – I2 = 0
• Applying Kirchhoff’s Voltage Law along the loop BCDB, we get
–11 (I1 – I2) + 6 (I – I1 + I2) + 12 I2 = 0
6I – 17 I1 + 28 I2 = 0
In the choice (b) the coefficient of I1 is wrong .
• Applying Kirchhoff’s Voltage Law along the loop ABCDA , we get
−12I1 − 11(I1 − I2 ) + 6(I − I1 + I2 ) + 12(I − I1 ) = 0
18I − 41I1 + 17 I2 = 0
• Applying Kirchhoff’s Voltage Law along the loop ABCEA , we get
−12 I1 − 11(I1 − I2 ) + 60 − 10 I = 0
10 I − 23I1 + 11I2 = 60

17. b, d
• The branch ADC has less resistance as compared to branch ABC . Therefore, the current (I – I1) > I1 and
thus , the voltage drop VA – VD > VA – VB . Hence . VB > VD
• VA = VC + E – I (10)
or VA= 0 + 100 – 10 I
 VA < 100 V

18. a, c, d
• In terms of potentials, the respective currents may be written as :
V − VB V − VD
I1 = A ; I2 = A
12 12
VD − VB VB − VC VB
I3 = ; I4 = =
12 11 11

and VA + 10 I – 100 = VC = 0
100 − VA
 I=
.10
• Applying Kirchhoff’s Current Law at B, we get
I1 + I3 = I4
VA − VB VD − VB VB
 + =
12 12 11
VA − VB VD − VB VB
or + = =0 Choice (a) is correct .
12 12 11

• Alternatively , the above equation may be expressed as


VB − VA VB − VD VB
+ + =0 Choice (c) is correct .
12 12 11
V − VB VB VB − VD
and A − = Choice (d) is correct .
12 11 12

19. b, d
Applying Kirchhoff’s Current Law at the junction A , we get
I1 + I2 = I
VA − VB VA − VD 100 − VA
+ =
12 12 10
Rearranging the above equation ,
VB − VA VD − VA 100 − VA
+ + =0 Choice (b) is correct .
12 12 10
VB − VA VD − VA VA − 100
and + + =0 Choice (d) is correct.
12 12 10

20. a, c
• Wheatstone bridge condition is satisfied .
RABRDC = RADRBC
• VB – VD cannot be equal to zero, because a battery is connected between B and D. Obviously,
VB – VD = 8V
• Current in the branch BD cannot be zero because Wheatstone Bridge condition is satisfied.

21. a, d
Since the condition of Wheatstone Bridge is satisfied with respect to
both the batteries, therefore , 8V battery cannot produce current at the
position of 12V battery, and vice versa.
• Thus, current in branch BD is only due to 8V battery.
• The direction of current is from D to B

22. a, d
With the same reason as stated above .
• The current in branch AC is only due to 12 V battery .
• The direction of current is from C to A .

23. a, d
Both the batteries are delivering current .

24. a, c
With respect to 8V to 12V batteries the circuit can be reoriented as shown in fig.
S 11.26 (A) and (B) .

25. a, b, d
Problems 26 to 28

26. a & b
Since the points 1, 2 and 3 have equal potential , therefore , R12 = R25 = R13 = 0

27. a & c
Due to symmetry
R
R01 = R02 = R03 =
3

28. a & b
• Since The resistance R12 , R23 and R31 has been shorted by the zero resistance wire, therefore, no current
flows through these three resistances.
• Due to symmetry equal current flows in R01 , R02 and R03 .

Problems 29 to 31

29. a
A tetrahedron is a completely symmetric figure. Between any two points, it can be reduced to a Whearstone
Bridge.

30. a, b, c & d
Note that the resistance CD overlaps the resistance AB .

31. a, b & c
• Due to the balanced Wheatstone Bridge condition , the potentials of the other two points are always equal.
• The branch between the points of same potential carries no current .
• Since the equivalent resistance between any two points are equal, therefore, same current comes out of the
battery in each case.

32. a & c
• The cells are connected in series because same current flows through each cell .
The current through the loop is
E + E2
I= 1
r1 + r2
Applying Kirchhoff’s law , we get
VA + Ea − Ir1 = VB
(E1 + E 2 )r1
or VB − VA = E a −
r1 + r2
E1r2 − E 2 r1
or VB − VA =
r1 + r2
• The difference VB – VA = 0 when E1r2 = E2r1
• The current in the loop flows even when VB – VA = 0 .

33. c & d
• The needle of a galvanometer diflects on either side and the zero is marked at its middle.
• Larger the deflection ; greater is the current .
• It does not measure current, it simply shows the presence of current .

34. a
A galvanometer G is converted into an ammeter by connecting a low
resistance shunt S in parallel with the galvanometer .

35. a, c & d
Ig
We know that IsS = IgG = S = G
Is
Since Is = I − Ig , therefore ,
 Ig 
S= G
 I − Ig 
 
Thus , S  G
1
S
I
S  Ig

36. a & d
• An ammeter is always connected in series with the branch through which current is to be measured.
• An ammeter measures exact value of current only when its resistance is equal to zero .

37. c
In a voltmeter, a high resistance is connected in series with the
galvanometer .

38. a, c & d
The magnitude of external resistance R is given by
V
R = −G
Ig
Thus , higher the value of G, lower is the value of R .
1
R , and
Ig
RV
39. a, b, c & d
All the statemtns are correct .

40. a, b, c & d
A potentiometer can be used to form a Wheatstone Bridge condition to measure an unknown resistance.

41. a
2
According to the given information, when a shunt S is connected across the galvanometer G, then th of
5
3
the total current passes through the galvanometer and th of the total current passes through the shunt.
5
Thus ,
G 3 3 3
= or G = S = (12) = 18
S 2 2 2

42. a & d
• Ea must be more than E, otherwise, balance point reading cannot be obtained .
• When external resistance is connected in parallel to E , then it delivers current and voltage across its
terminal is less than its emf , therefore , 0  .

43. b
E = E0 0
where L is the total length of the potentiometer
L
 75 
thus E = 2   = 1.5 V
 100 

44. d
According to the given information
E 0
E
IR  where I =
R+r
Since R and r both are unknown, therefore , data is insufficient .

45. a
The internal resistance
 
r = R  0 − 1
 
 75 
or r = 10  − 1 = 2.5
 60 

Problems 46 to 49

46. a, b, c & d
• Applying KCL at junction A
I = I1 + I2  I3 = I – I1
• Applying KCL at junction B
I1 = I3 + I4
• Since I4 = I2 and I5 = I1 due to diagonal symmetry .
 I1 = I3 + I2 .

47. b & c
Obviously, due to diagonal symmetry
VA – VD = VB – VC
and VA – VB = VD – VC
48. b, c & d
• When r1 > r2 , then I2 > I1 and since I3 = I1 – I2 therefore, I3 becomes negative.
Thus , current in the branch BD flows from D to B , hence VB – VD < 0 .
• Since VA – VB = VD + VD
 VA + VC = VB + VD
• Since VA – VC = E
 VB + VD = VA + VC = VA + (VA – E)
or VB + VD = 2VA – E

49. b
2r1r2
If r0 = 0, the equivalent resistance becomes R =
r1 + r2
r1 + r2
If r0 =  , then the equivalent resistance becomes R =
2
(r1 + r2 )r0 + 2r1r2
Thus , the correct value of equivalent resistance is R =
2r0 + r1 + r2

50. b & c
In series combination of capacitors
• potential difference is inversely proportional to capacitance
• energy stored in also inversely proportional to resistance.

51. a & c
In parallel combination of capacitors
• charge is stored in direct proportion to capacitance .
• energy is stored in direct proportion to capacitance.

52. a, b & d
Potential energy stored in the capacitor is partly lost into heat in the form of electromagnetic waves .

53. c
The amount of sharing of charge is directly proportional to capacitance.

54. b & c
• The energy lost by the charged capacitor is always more than the energy gained by the uncharged capacitor
because part of the energy is lost as heat .
• The capacitors share energy is proportion to their capacitance.

55. b
Uloss Cunch arg ed
=
Ui Cchared + Cunch arg ed

Problems 56 to 59

56. b & c
• The left plate of each 4F capacitor is positive .
• Applying KVL along ABDA , we get
q  q −q  q
− 2 − 2 1 + 1 = 0
4  2  2
4
or q 2 = q1
3
q2 − q2
Thus, VB − VD = 0
2

57. a , b, c & d
• If VC = 0 , then VA = + 20 V
• Applying KCL at the junction B, we get
4(VA − VB ) + 2(VD − VB ) + 2(0 − VB ) = 0
or 4(VA − VB ) + 2(VD − VB ) = 2VB (1)
• Applying KCL at the junction D, we get
2(VA − VD ) + 2(VB − VD ) + 4(0 − VD ) = 0
or 2(VA − VD ) + 2(VB − VD ) = 4VD (2)
• Since VA – VB = VD – VC
 VA + VC = VB + VD
or VA = VB + VD ( VC = 0)

58. b & c
After solving the equation (1) and (2) , we get VB = 12V and VD = 8V

59. c
q1 = (VA − VB )4
= (20 – 12) 4 = 32C
q2 = (VA – VD) 2
= (20 – 8) 2 = 24 C
q3 = q1 – q2 = 8C.

Problems 60 to 61
60. b
• As the switch is closed, no current flows through R2 , therefore, voltage drop across it is zero .
• With the passage of time current in the circuit decreases but it does not becomes equal to zer o .
• Initially, the value of current is maximum .

61. b , c & d
A long time after closing the switch
E R2
• voltage drop across the capacitor is
R1 + R 2
E
• the current through the battery is
R1 + R 2
• the current through the capacitor becomes zero .
2
1  E R2 
• the energy stored in the capacitor is U = C  
2  R1 + R 2 

You might also like